Skip to main content
Top
Published in: Journal of Inequalities and Applications 1/2018

Open Access 01-12-2018 | Research

Majorization involving the cyclic moving average

Authors: Tao Zhang, Huan-Nan Shi, Bo-Yan Xi, Alatancang Chen

Published in: Journal of Inequalities and Applications | Issue 1/2018

Activate our intelligent search to find suitable subject content or patents.

search-config
download
DOWNLOAD
print
PRINT
insite
SEARCH
loading …

Abstract

We solve an open problem on some majorization inequalities involving the cyclic moving average.
Notes

Publisher’s Note

Springer Nature remains neutral with regard to jurisdictional claims in published maps and institutional affiliations.

1 Introduction

We first recall two definitions.
Definition 1.1
([1])
For fixed \(n\geq 2\), let \(x=(x_{1},x_{2},\ldots ,x_{n})\) and \(y=(y_{1},y_{2},\ldots ,y_{n})\) be two n-tuples of real numbers.
(i)
x is said to be majorized by y (in symbols, \(x \prec y\)) if
$$ \sum_{i = 1}^{k} x_{[i]} \le \sum _{i = 1}^{k} y_{[i]}\quad\mbox{for }k = 1,2,\ldots ,n -1,\quad \mbox{and} \quad \sum_{i = 1}^{n} x_{i} = \sum_{i = 1}^{n} y_{i}, $$
where \(x_{[1]}\ge \cdots \ge x_{[n]}\) and \(y_{[1]}\ge \cdots \ge y _{[n]}\) are rearrangements of x and y in descending order.
 
(ii)
Let \(\Omega \subset \mathbb{R}^{n}\). A function \(\varphi :\Omega \rightarrow \mathbb{R}\) is said to be a Schur-convex function (shortly, an S-convex function) if
$$ x \prec y \Rightarrow \varphi (x) \leq \varphi (y). $$
 
For example:
$$\begin{aligned}& a^{(2)}= \biggl( \frac{a_{1}+a_{2}}{2},\frac{a_{2}+a_{3}}{2},\ldots , \frac{a _{n}+a_{1}}{2} \biggr) , \\& a^{(3)}= \biggl( \frac{a_{1}+a_{2}+a_{3}}{3},\frac{a_{2}+a_{3}+a_{4}}{3}, \ldots , \frac{a_{n}+a_{1}+a_{2}}{3} \biggr) . \end{aligned}$$
In 2006, I. Olkin, one of the authors of the book [1], wrote a letter to K. Z. Guan, referring to the following interesting question: is it true that
$$ a^{(k+1)}\prec a^{(k)},\quad 1\leq k\leq n-1? $$
(1)
However, a proof for \(a^{(k+1)}\prec a^{(k)}\) remains elusive (see [1], p. 63).
In 2010, Shi [2] proved that (1) holds when \(n= 4\), \(k= 2 \) and \(n=5\), \(k=3 \). In this paper, we prove that (1) holds for any \(n\geq 2\) and \(1\leq k\leq n-1\).
For any \(1\leq k\leq n\), let
$$ a^{(k)}_{[1]}\geq a^{(k)}_{[2]}\geq \cdots \geq a^{(k)}_{[n]} $$
be the ordered component of the sequence \(a^{(k)}_{1},a^{(k)}_{2}, \ldots ,a^{(k)}_{n}\). We denote
$$ S_{h}^{(k)}=\sum _{i=1}^{h}a^{(k)}_{[i]}, \quad 1\leq h\leq n. $$

2 Lemmas and corollaries

For proving our main results, we need the following lemmas.
Lemma 2.1
Let \(n\geq 2\) and \(1\le k\le n-1\). Then
$$ \textstyle\begin{cases} a^{(k)}_{1}\geq a^{(k)}_{2}\geq \cdots \geq a^{(k)}_{n-k+1}, \\ a^{(k)}_{n}\geq a^{(k)}_{n-1}\geq \cdots \geq a^{(k)}_{n-k+1}, \\ a^{(k)}_{1}\geq a^{(k)}_{n}. \end{cases} $$
(2)
Proof
For any \(1\le k\le n-1\) and \(1\le i\le n\), we have
$$ k\bigl(a_{i}^{(k)}-a_{i+1}^{(k)}\bigr)= ( a_{i}+a_{i+1}+\cdots +a_{i+k-1} ) - ( a_{i+1}+a_{i+2}+\cdots +a_{i+k} ) =a_{i}-a_{i+k}. $$
Note that \(a_{1}\geq a_{2}\geq \cdots \geq a_{n}\), so we can induce that
(1)
if \(i+k\leq n\), that is, \(1\leq i\leq n-k\), then \(a_{i}\geq a_{i+k}\). It follows that \(a_{i}^{(k)}\geq a_{i+1}^{(k)}\).
 
(2)
if \(i+k> n\), that is, \(n-k< i\leq n\), then \(a_{i+k}=a_{n+(i+k-n)}=a _{i+k-n}\). Since \(1\le k\le n-1\), we have \(n\geq i> i+k-n\). So \(a_{i}\geq a_{i+k-n}=a_{i+k}\). It follows that \(a_{i}^{(k)}\geq a_{i+1} ^{(k)}\).
 
(3)
\(k(a_{1}^{(k)}-a_{n}^{(k)})= ( a_{1}+a_{2}+\cdots +a_{k} ) - ( a_{n}+a_{1}+\cdots +a_{k-1} ) =a_{k}-a_{n}\geq 0\). Therefore (2) holds.
 
 □
From the proof of Lemma 2.1 it is easy to deduce the following:
Corollary 2.2
Let \(n\geq 2\), \(1\leq k\leq n-1\), and let \(\sum_{i=0}^{-1}a^{(k)}_{n-i}=0\). For any \(1\leq h\leq n-1\), there exist \(1\leq h_{1}\leq n-k\) and \(-1\leq h_{2}\leq k-2\) such that \(h=h_{1}+h _{2}+1\) and
$$ S_{h}^{(k)}= \sum_{i=1}^{h_{1}}a^{(k)}_{i}+ \sum_{i=0}^{h_{2}}a^{(k)} _{n-i}. $$
Lemma 2.3
Let \(n\geq 4\), \(2\leq k\leq n-2\), and \(0\leq r\leq k-2\).
(i)
If
$$ a^{(k)}_{2}\leq a^{(k)}_{n-r} \leq a^{(k)}_{1}, $$
(3)
then
$$ a^{(k+1)}_{2}\leq a^{(k+1)}_{n-r} \leq a^{(k+1)}_{1}. $$
 
(ii)
If
$$ a^{(k)}_{n-k+1}\leq a^{(k)}_{n-r} \leq a^{(k)}_{n-k}, $$
(4)
then
$$ a^{(k+1)}_{n-k}\leq a^{(k+1)}_{n-r-1} \leq a^{(k+1)}_{n-k-1}. $$
 
(iii)
For \(2\leq m\leq n-k-1\), if
$$ a^{(k)}_{m+1}\leq a^{(k)}_{n-r} \leq a^{(k)}_{m}, $$
(5)
then
$$ a^{(k+1)}_{m+1}\leq a^{(k+1)}_{n-r}, \qquad a^{(k+1)}_{n-r-1}\leq a^{(k+1)}_{m-1}. $$
(6)
 
(iv)
If
$$ a^{(k)}_{n}\leq a^{(k)}_{n-k} , $$
(7)
then
$$ a^{(k+1)}_{n-1}\leq a^{(k+1)}_{n-k-1} . $$
 
(v)
For \(2\leq m\leq n-k-1\), if
$$ a^{(k)}_{n}\leq a^{(k)}_{m} , $$
(8)
then
$$ a^{(k+1)}_{n-1}\leq a^{(k+1)}_{m} . $$
 
(vi)
For \(2\leq m\leq n-k\), if
$$ a^{(k)}_{n-r-1}\leq a^{(k)}_{m} \leq a^{(k)}_{n-r}, $$
(9)
then
$$ a^{(k+1)}_{n-r-2}\leq a^{(k+1)}_{m-1}, \qquad a^{(k+1)}_{m}\leq a^{(k+1)}_{n-r}. $$
 
Proof
(i)
By Lemma 2.1 we have
$$ \textstyle\begin{cases} a^{(k+1)}_{1}\geq a^{(k+1)}_{2}\geq \cdots \geq a^{(k+1)}_{n-k}, \\ a^{(k+1)}_{n}\geq a^{(k+1)}_{n-1}\geq \cdots \geq a^{(k+1)}_{n-k}, \\ a^{(k+1)}_{1}\geq a^{(k+1)}_{n}. \end{cases} $$
It follows that
$$ a^{(k+1)}_{1}=\max \bigl\{ a^{(k+1)}_{1}, a^{(k+1)}_{2},\ldots , a^{(k+1)} _{n}\bigr\} . $$
Thus we have
$$ a^{(k+1)}_{n-r}\leq a^{(k+1)}_{1}. $$
By the left inequality of (3) we have
$$ a_{2}+ \cdots + a_{k+1}\leq a_{n-r}+ \cdots + a_{n-r+k-1}. $$
Note that \(a_{k+2}\leq a_{k-r}=a_{n+k-r}\), so we have
$$ a_{2}+ \cdots + a_{k+1}+ a_{k+2}\leq a_{n-r}+\cdots + a_{n-r+k-1}+ a _{n+k-r}. $$
Therefore
$$ a^{(k+1)}_{2}\leq a^{(k+1)}_{n-r}. $$
 
(ii)
By Lemma 2.1 we have
$$ a^{(k+1)}_{n-k}\leq a^{(k+1)}_{n-r-1}. $$
By the right inequality of (4) we get
$$ a_{n-r} +\cdots + a_{n-r+k-1}\leq a_{n-k} +\cdots +a_{n-1}. $$
Note that \(a_{n-r-1}\leq a_{n-k-1}\), so we have
$$ a_{n-r} +\cdots + a_{n-r+k-1} + a_{n-r-1}\leq a_{n-k} +\cdots +a_{n-1}+ a_{n-k-1}. $$
This means that
$$ a^{(k+1)}_{n-r-1}\leq a^{(k+1)}_{n-k-1}. $$
 
(iii)
By (5) we get
$$ a_{m+1}+ \cdots + a_{m+k}\leq a_{n-r} + \cdots + a_{n-r+k-1} \leq a _{m}+ \cdots +a_{m+k-1}. $$
Note that \(n\geq m+k+1\geq k-r \geq 1\) and \(n\geq n-r-1\geq m-1 \geq 1\), so we have
$$ a_{m+k+1}\leq a_{n-r+k}=a_{k-r},\qquad a_{n-r-1} \leq a_{m-1}. $$
It follows that
$$ a_{m+1}+ \cdots + a_{m+k} + a_{m+k+1}\leq a_{n-r} + \cdots + a_{n-r+k-1} + a_{n-r+k} $$
and
$$ a_{n-r-1} + a_{n-r} + \cdots + a_{n-r+k-1} \leq a_{m-1} +a_{m}+ \cdots +a_{m+k-1}. $$
Therefore (6) holds.
 
(iv)
By (7) we get
$$ a_{n} + \cdots + a_{n+k-1}\leq a_{n-k} + \cdots + a_{n-1}. $$
Since \(a_{n-1}\leq a_{n-k-1}\), we have
$$ a_{n} + \cdots + a_{n+k-1} + a_{n-1}\leq a_{n-k} + \cdots + a_{n-1} + a_{n-k-1}. $$
It follows that
$$ a^{(k+1)}_{n-1}\leq a^{(k+1)}_{n-k-1}. $$
 
(v)
By (8) we have
$$ a_{n} + \cdots + a_{n+k-1}\leq a_{m} +\cdots + a_{m+k-1}. $$
Note that \(n-1\geq m+k\) and \(a_{n-1}\leq a_{m+k}\), so we have
$$ a_{n} + \cdots + a_{n+k-1}+ a_{n-1}\leq a_{m} +\cdots + a_{m+k-1}+ a _{m+k}. $$
This means that
$$ a^{(k+1)}_{n-1}\leq a^{(k+1)}_{m}. $$
 
(vi)
By (9) we get
$$ a_{n-r-1}+\cdots + a_{n-r+k-2}\leq a_{m} +\cdots + a_{m+k-1}\leq a _{n-r}+\cdots + a_{n-r+k-1}. $$
Note that \(0\leq r\leq k-2\) and \(2\leq m\leq n-k\), so we have
$$ n\geq n-r-2\geq n-k\geq m\geq m-1\geq 1. $$
It follows that
$$ a_{n-r-2}\leq a_{m-1}. $$
So we get
$$ a_{n-r-2} + a_{n-r-1}+\cdots + a_{n-r+k-2}\leq a_{m-1} + a_{m} + \cdots + a_{m+k-1}. $$
Therefore
$$ a^{(k+1)}_{n-r-2}\leq a^{(k+1)}_{m-1}. $$
Note that \(m\leq n-k\), so we have
$$ k-r\leq m+k\leq n,\qquad a_{m+k}\leq a_{k-r}= a_{n+k-r}. $$
It follows that
$$ a_{m} +\cdots + a_{m+k-1} + a_{m+k}\leq a_{n-r}+\cdots + a_{n-r+k-1} + a_{n+k-r}. $$
Therefore
$$ a^{(k+1)}_{m}\leq a^{(k+1)}_{n-r}. $$
 
 □
Lemma 2.4
Let \(n\geq 4\), \(2\leq k\leq n-1\), \(1\leq m\leq n-k\), and \(0\leq r \leq k-2\).
(i)
If \(a^{(k)}_{m+1}\leq a^{(k)}_{n-r}\leq a^{(k)}_{m}\), then
$$ S_{m+r+1}^{(k)}= \sum _{i=1}^{m}a^{(k)}_{i}+\sum _{i=0}^{r}a^{(k)}_{n-i}. $$
(10)
 
(ii)
If \(a^{(k)}_{n}\leq a^{(k)}_{m} \), then
$$ S_{m}^{(k)}= \sum_{i=1}^{m}a^{(k)}_{i}. $$
 
(iii)
For \(2\leq m\leq n-k\), if \(a^{(k)}_{n-r-1}\leq a^{(k)} _{m}\leq a^{(k)}_{n-r}\), then
$$ S_{m+r+1}^{(k)}= \sum_{i=1}^{m}a^{(k)}_{i}+ \sum_{i=0}^{r}a^{(k)}_{n-i}. $$
 
Proof
We only prove (i). Using a similar method, we can obtain (ii) and (iii).
By Lemma 2.1 we have
$$ \textstyle\begin{cases} a_{1}^{(k)}\geq a_{2}^{(k)}\geq \cdots \geq a^{(k)}_{m} \geq a^{(k)} _{n-r}\geq a^{(k)}_{m+1}, \\ a^{(k)}_{n}\geq a^{(k)}_{n-1}\geq \cdots \geq a^{(k)}_{n-r+1}\geq a ^{(k)}_{n-r}\geq a^{(k)}_{m+1}. \end{cases} $$
(11)
By Corollary 2.2 we let
$$ S_{m+r+1}^{(k)}=\sum_{i=1}^{h_{1}}a^{(k)}_{i}+ \sum_{i=0}^{h_{2}}a^{(k)} _{n-i}, $$
(12)
where \(1\leq h_{1}\leq n-k+1\), \(-1\leq h_{2}\leq k-2\), and \(\sum_{i=0} ^{-1}a^{(k)}_{n-i}=0\). It is clear that
$$ m+r+1= h_{1}+h_{2}+1. $$
(13)
Next, we prove that \(h_{1}=m\) and \(h_{2}=r\).
(1)
If \(h_{1}\geq m+1\), which means that the right-hand side of (12) includes \(a^{(k)}_{m+1}\), then by (11) the right-hand side of (12) should include \(a^{(k)}_{1},a^{(k)} _{2},\ldots ,a^{(k)}_{m+1}\) and \(a^{(k)}_{n-r}, a^{(k)}_{n-r+1}, \ldots , a^{(k)}_{n}\), so we have
$$ h_{1}+h_{2}+1\geq m+r+2>m+r+1. $$
This is a contradiction with (13).
 
(2)
If \(h_{1}\leq m-1\), then by (13) we have \(k-2\geq h_{2} \geq r+1\). Together with (11), we get
$$ a^{(k)}_{n-h_{2}}\leq a^{(k)}_{n-r-1}\leq a^{(k)}_{n-r}\leq a^{(k)} _{m}. $$
So the right-hand side of (12) must include \(a_{m}^{(k)}\), which means that \(h_{1}\geq m\). This is a contradiction with \(h_{1}\leq m-1\).
 
Therefore \(h_{1}=m\) and \(h_{2}=r\). So (10) holds. □
Corollary 2.5
Let \(n\geq 4\), \(2\leq k\leq n-2\), \(1\leq m\leq n-k\), and \(0\leq r \leq k-2\), and let
$$ a^{(k)}_{m+1}\leq a^{(k)}_{n-r} \leq a^{(k)}_{m}. $$
(14)
(i)
For \(m=1\), we have
$$ S_{r+2}^{(k+1)}=a^{(k+1)}_{1}+ \sum_{i=0}^{r}a^{(k+1)}_{n-i}. $$
(15)
 
(ii)
For \(m=n-k\), we have
$$ S_{n-k+r+1}^{(k+1)}= \sum _{i=1}^{n-k-1}a^{(k+1)}_{i}+ \sum _{i=0}^{r+1}a^{(k+1)}_{n-i}. $$
(16)
 
(iii)
For \(2\leq k\leq n-3\) and \(2\leq m\leq n-k-1\), \(S_{m+r+1}^{(k+1)}\) must be one of the following two cases:
$$ S_{m+r+1}^{(k+1)}=\sum_{i=1}^{m}a^{(k+1)}_{i}+ \sum_{i=0}^{r}a^{(k+1)} _{n-i}, $$
or
$$ S_{m+r+1}^{(k+1)}=\sum_{i=1}^{m-1}a^{(k+1)}_{i}+ \sum_{i=0}^{r+1}a^{(k+1)} _{n-i}. $$
 
Proof
(i)
If \(m=1\), by (14) we have
$$ a_{2}^{(k)}\leq a^{(k)}_{n-r}\leq a_{1}^{(k)}. $$
By Lemma 2.3(i) we have
$$ a^{(k+1)}_{2}\leq a^{(k+1)}_{n-r}\leq a^{(k+1)}_{1}, $$
and then by Lemma 2.4(i) we can induce that (15) holds.
 
(ii)
If \(m=n-k\), then by (14) we have
$$ a^{(k)}_{n-k+1}\leq a^{(k)}_{n-r}\leq a^{(k)}_{n-k}. $$
By Lemma 2.3(ii) we have
$$ a^{(k+1)}_{n-k}\leq a^{(k+1)}_{n-r-1}\leq a^{(k+1)}_{n-k-1}, $$
and then by Lemma 2.4(i) we can induce that (16) holds.
 
(iii)
By Corollary 2.2 we let
$$ S_{m+r+1}^{(k+1)}=\sum_{i=1}^{p}a^{(k+1)}_{i}+ \sum_{i=0}^{q}a^{(k+1)} _{n-i}, $$
(17)
where \(1\leq p\leq n-k\), \(-1\leq q\leq k-2\), and \(\sum_{i=0}^{-1}a^{(k)} _{n-i}=0\). Then we have
$$ p+q+1= m+r+1. $$
(18)
 
Next, we prove that \(p=m\) or \(p=m-1\).
(1)
If \(p\geq m+1\), then by Lemma (2.3)(iii) we have \(a^{(k+1)}_{m+1}\leq a^{(k+1)}_{n-r}\). So we get
$$ a^{(k+1)}_{p}\leq a^{(k+1)}_{m+1}\leq a^{(k+1)}_{n-r}. $$
Thus the right-hand side of (17) includes \(a^{(k+1)}_{n-r}\), which means that \(q \geq r\). Therefore
$$ p+q+1\geq m+r+2> m+r+1. $$
This is a contradiction with (18).
 
(2)
If \(1\leq p\leq m-2\), then by (18) we have \(n-q\leq n-r-2\). By Lemma 2.3(iii) we get \(a^{(k+1)}_{n-r-1}\leq a^{(k+1)} _{m-1}\). It follows that
$$ a^{(k+1)}_{n-q}\leq a^{(k+1)}_{n-r-2}\leq a^{(k+1)}_{n-r-1}\leq a^{(k+1)} _{m-1}. $$
So the right-hand side of (17) must include \(a^{(k+1)}_{m-1}\). Therefore
$$ p \geq m-1. $$
This is a contradiction with \(1\leq p\leq m-2\).
 
Thus \(p=m\) or \(p=m-1\). □
In a similar way as in Corollary 2.5, we can prove the following corollaries.
Corollary 2.6
Let \(n\geq 4\), \(2\leq k\leq n-2\), \(2\leq m\leq n-k\), and \(0\leq r \leq k-2\).
(i)
If \(a^{(k)}_{n}\leq a^{(k)}_{m} \), then \(S_{m}^{(k+1)}\) must be one of the following two cases:
$$ S_{m}^{(k+1)}= \sum_{i=1}^{m}a^{(k+1)}_{i} $$
or
$$ S_{m}^{(k+1)}=\sum_{i=1}^{m-1}a^{(k+1)}_{i}+a^{(k+1)}_{n}. $$
 
(ii)
If \(a^{(k)}_{n-r-1}\leq a^{(k)}_{m}\leq a^{(k)}_{n-r}\), then \(S_{m+r+1}^{(k+1)}\) must be one of the following two cases:
$$ S_{m+r+1}^{(k+1)}=\sum_{i=1}^{m}a^{(k+1)}_{i}+ \sum_{i=0}^{r}a^{(k+1)} _{n-i} $$
or
$$ S_{m+r+1}^{(k+1)}=\sum_{i=1}^{m-1}a^{(k+1)}_{i}+ \sum_{i=0}^{r+1}a^{(k+1)} _{n-i}. $$
 
Corollary 2.7
Let \(n\geq 4\), \(2\leq k\leq n-2\), \(1\leq m\leq n-k\), and \(-1\leq r \leq k-2\), and let \(\sum_{i=0}^{-1}a^{(k)}_{n-i}=0\). If
$$ S_{m+r+1}^{(k)}= \sum _{i=1}^{m}a^{(k)}_{i}+\sum _{i=0}^{r}a^{(k)}_{n-i}, $$
then we have:
(i)
if \(m=1\), then
$$ S_{r+2}^{(k+1)}=a^{(k+1)}_{1}+ \sum_{i=0}^{r}a^{(k+1)}_{n-i}; $$
 
(ii)
if \(2\leq m\leq n-k\), then \(S_{m+r+1}^{(k+1)}\) must be one of the following two cases:
$$ S_{m+r+1}^{(k+1)}=\sum_{i=1}^{m}a^{(k+1)}_{i}+ \sum_{i=0}^{r}a^{(k+1)} _{n-i} $$
or
$$ S_{m+r+1}^{(k+1)}=\sum_{i=1}^{m-1}a^{(k+1)}_{i}+ \sum_{i=0}^{r+1}a^{(k+1)} _{n-i}. $$
 
Lemma 2.8
Let \(n\geq 4\), \(2\leq k\leq n-2\), \(1\leq m\leq n-k\), and \(-1\leq r \leq k-2\), and let \(\sum_{i=0}^{-1}a^{(k)}_{n-i}=0\). If
$$ \textstyle\begin{cases} S_{m+r+1}^{(k)}= \sum_{i=1}^{m}a^{(k)}_{i}+\sum_{i=0}^{r}a^{(k)}_{n-i}, \\ S_{m+r+1}^{(k+1)}= \sum_{i=1}^{m}a^{(k+1)}_{i}+\sum_{i=0}^{r}a^{(k+1)}_{n-i}, \end{cases} $$
(19)
then
$$ S_{m+r+1}^{(k)}\geq S_{m+r+1}^{(k+1)}. $$
(20)
Proof
By a simple calculation we obtain
$$ S_{m+r+1}^{(k)}-S_{m+r+1}^{(k+1)}= \frac{1}{k+1} \Biggl( S_{m+r+1}^{(k)}- \sum _{i=k-r}^{k+m}a_{i} \Biggr) . $$
By (19) we have
$$ a^{(k)}_{m+1}\leq a^{(k)}_{n-r}. $$
It follows that
$$ a_{m+1}+a_{m+2}+\cdots +a_{k+m}\leq a_{n-r}+a_{n-r+1}+\cdots +a_{n-r+k-1}. $$
So we have
$$ a_{k-r}+a_{k-r+1}+\cdots +a_{k+m}\leq a_{n-r}+a_{n-r+1}+\cdots +a_{n+m}. $$
Note that
$$ \textstyle\begin{cases} a_{n-r}+a_{n-r+1}+\cdots +a_{n+m}\leq a_{n-r+j}+a_{n-r+1+j}+\cdots +a _{n+m+j},& 0\leq j\leq r, \\ a_{k-r}+a_{k-r+1}+\cdots +a_{k+m}\leq a_{n-r+j}+a_{n-r+1+j}+\cdots +a _{n+m+j}, &r+1\leq j\leq k-1. \end{cases} $$
Thus we can induce that
$$ S_{m+r+1}^{(k)}= \sum _{i=1}^{m}a^{(k)}_{i}+\sum _{i=0}^{r}a^{(k)}_{n-i}= \frac{1}{k}\sum_{i=n-r}^{n+m}\sum _{j=0}^{k-1} a_{j+i} = \frac{1}{k} \sum_{j=0}^{k-1}\sum _{i=n-r}^{n+m}a_{j+i} \geq \sum _{i=k-r}^{k+m}a _{i}. $$
This means that (20) holds. □
Lemma 2.9
Let \(n\geq 4\), \(2\leq k\leq n-2\), \(2\leq m\leq n-k\), and \(-1\leq r \leq k-2\), and let \(\sum_{i=0}^{-1}a^{(k)}_{n-i}=0\). If
$$ \textstyle\begin{cases} S_{m+r+1}^{(k)}= \sum_{i=1}^{m}a^{(k)}_{i}+\sum_{i=0}^{r}a^{(k)}_{n-i}, \\ S_{m+r+1}^{(k+1)}= \sum_{i=1}^{m-1}a^{(k+1)}_{i}+\sum_{i=0}^{r+1}a^{(k+1)}_{n-i}, \end{cases} $$
(21)
then
$$ S_{m+r+1}^{(k)}\geq S_{m+r+1}^{(k+1)}. $$
(22)
Proof
Note that
$$\begin{aligned} &S_{m+r+1}^{(k)}-S_{m+r+1}^{(k+1)} \\ &\quad = \frac{1}{k+1} \Biggl( \sum _{i=1}^{m-1}a^{(k)}_{i}+ \sum _{i=0}^{r}a^{(k)}_{n-i} \Biggr) +a^{(k)}_{m}-a^{(k+1)}_{n-r-1}- \frac{1}{k+1} \sum _{i=k-r}^{k+m-1}a_{i} \\ &\quad = \frac{1}{k+1} \Biggl( \sum _{i=1}^{m}a^{(k)}_{i}+ \sum _{i=0}^{r}a^{(k)}_{n-i}- \sum _{i=n-r-1}^{n+m-1}a_{i} \Biggr) \\ &\quad =\frac{1}{k+1} \Biggl( S_{m+r+1}^{(k)}-\sum _{i=n-r-1}^{n+m-1}a _{i} \Biggr) . \end{aligned}$$
By (21) we have
$$ a^{(k)}_{n-r-1}\leq a^{(k)}_{m}. $$
It follows that
$$ a_{n-r-1}+a_{n-r}+\cdots +a_{n-r+k-2}\leq a_{m}+a_{m+1}+\cdots +a_{m+k-1}. $$
So we can induce that
$$ a_{n-r-1}+a_{n-r}+\cdots +a_{n+m-1}\leq a_{k-r-1}+a_{k-r}+\cdots +a _{k+m-1}. $$
Since
$$ \textstyle\begin{cases} a_{n-r-1}+a_{n-r}+\cdots +a_{n+m-1}\leq a_{n-r+j}+a_{n-r+1+j}+\cdots +a_{n+m+j}, &0\leq j\leq r, \\ a_{k-r-1}+a_{k-r}+\cdots +a_{k+m-1}\leq a_{n-r+j}+a_{n-r+1+j}+\cdots +a_{n+m+j}, &r+1\leq j\leq k-1, \end{cases} $$
we have
$$ S_{m+r+1}^{(k)} =\frac{1}{k} \sum _{j=0}^{k-1}\sum_{i=n-r}^{n+m}a_{j+i} \geq \sum_{i=n-r-1}^{n+m-1}a_{i}. $$
This means that (22) holds. □

3 Main results

We are now in a position to prove our main results (1) in two cases: \(k=1\) and \(2 \leq k\leq n-1\).
Theorem 3.1
For any \(n\geq 2\), we have
$$ a^{(2)}\prec a^{(1)}=a. $$
(23)
Proof
It is clear that (23) holds if \(n=2\). Next, let \(n\geq 3\). Then we have
$$ \frac{a_{1}+a_{2}}{2}= S_{1}^{(2)}\leq S_{1}^{(1)}=a_{1}, \qquad S_{n}^{(2)}=S _{n}^{(1)}. $$
For \(2\leq m\leq n-1\), we prove that \(S_{m}^{(2)}\leq S_{m}^{(1)}\) in the following two cases:
(i)
If \(S_{m}^{(2)}= \sum_{i=1}^{m}a^{(2)}_{i}\), then
$$ S_{m}^{(2)}-S_{m}^{(1)}= \frac{a_{m+1}-a_{1}}{2}\leq 0. $$
 
(ii)
If \(S_{m}^{(2)}= a^{(2)}_{n}+\sum_{i=1}^{m-1}a^{(2)}_{i}\), then
$$ S_{m}^{(2)}-S_{m}^{(1)}= \frac{a_{n}-a_{m}}{2}\leq 0. $$
So (23) holds.
 
 □
Theorem 3.2
For any \(n\geq 3\) and \(2\leq k\leq n-1\), we have
$$ a^{(k+1)}\prec a^{(k)}. $$
(24)
Proof
It is clear that (24) holds for any \(n\geq 3\), \(k=n-1\) and for \(n= 3\), \(k=1\).
Next, let \(n\geq 4\) and \(2\leq k\leq n-2\).
For any \(1\leq m\leq n-k\) and \(-1\leq r\leq k-2\), let \(\sum_{i=0}^{-1}a ^{(k)}_{n-i}=0\), and let
$$ S_{m+r+1}^{(k)}= \sum_{i=1}^{m}a^{(k)}_{i}+ \sum_{i=0}^{r}a^{(k)}_{n-i}. $$
Next, we prove that
$$ S_{m+r+1}^{(k)}\geq S_{m+r+1}^{(k+1)} $$
in the following two cases:
(i)
If \(m=1\) and \(-1\leq r\leq k-2\), then by Corollary 2.7(i) and Lemma 2.8 we get
$$ S_{r+2}^{(k)}\geq S_{r+2}^{(k+1)}. $$
 
(ii)
If \(2\leq m\leq n-k\) and \(-1\leq r\leq k-2\), then by Corollary 2.7(ii), Lemma 2.8, and Lemma 2.9 we get
$$ S_{m+r+1}^{(k)}\geq S_{m+r+1}^{(k+1)}. $$
Note that
$$ S_{n}^{(k)}= S_{n}^{(k+1)}, $$
so (24) holds.
 
 □

4 Discussion

In the theory of majorizations, there are two key concepts, majorizing relations and Schur-convex functions. Majorizing relations are weaker ordered relations among vectors, and Shur-convex functions are an extension of classical convex functions. Combining these two objects is an effective method of constructing inequalities.
In the theory of majorization, there are two important and fundamental objects, establishing majorizing relations among vectors and finding various Schur-convex functions. Majorizing relations deeply characterize intrinsic connections among vectors, and combining a new majorizing relation with suitable Schur-convex functions can lead to various interesting inequalities; see [313].

Competing interests

The authors declare that they have no competing interests.
Open Access This article is distributed under the terms of the Creative Commons Attribution 4.0 International License (http://creativecommons.org/licenses/by/4.0/), which permits unrestricted use, distribution, and reproduction in any medium, provided you give appropriate credit to the original author(s) and the source, provide a link to the Creative Commons license, and indicate if changes were made.

Publisher’s Note

Springer Nature remains neutral with regard to jurisdictional claims in published maps and institutional affiliations.
Literature
1.
go back to reference Marshall, A.W., Olkin, I., Arnord, B.C.: Inequalities: Theory of Majorization and Its Application, 2nd edn. Springer, New York (2011) CrossRef Marshall, A.W., Olkin, I., Arnord, B.C.: Inequalities: Theory of Majorization and Its Application, 2nd edn. Springer, New York (2011) CrossRef
2.
go back to reference Shi, H.N.: A class of majorization inequality with applications. J. Beijing Union Univ. (Nat. Sci.) 24(1), 60–64 (2010) Shi, H.N.: A class of majorization inequality with applications. J. Beijing Union Univ. (Nat. Sci.) 24(1), 60–64 (2010)
3.
go back to reference Wu, S.H., Shi, H.N.: A relation of weak majorization and its applications to certain inequalities for means. Math. Slovaca 61(4), 561–570 (2011) MathSciNetCrossRefMATH Wu, S.H., Shi, H.N.: A relation of weak majorization and its applications to certain inequalities for means. Math. Slovaca 61(4), 561–570 (2011) MathSciNetCrossRefMATH
4.
go back to reference Shi, H.N., Jiang, Y.M., Jiang, W.D.: Schur-convexity and Schur-geometrically concavity of Gini means. Comput. Math. Appl. 57, 266–274 (2009) MathSciNetCrossRefMATH Shi, H.N., Jiang, Y.M., Jiang, W.D.: Schur-convexity and Schur-geometrically concavity of Gini means. Comput. Math. Appl. 57, 266–274 (2009) MathSciNetCrossRefMATH
5.
go back to reference Shi, H.N., Wu, S.H., Qi, F.: An alternative note on the Schur-convexity of the extended mean values. Math. Inequal. Appl. 9, 219–224 (2006) MathSciNetMATH Shi, H.N., Wu, S.H., Qi, F.: An alternative note on the Schur-convexity of the extended mean values. Math. Inequal. Appl. 9, 219–224 (2006) MathSciNetMATH
8.
go back to reference Shi, H.N., Zhang, J.: Compositions involving Schur harmonically convex functions. J. Comput. Anal. Appl. 22(5), 907–922 (2017) MathSciNet Shi, H.N., Zhang, J.: Compositions involving Schur harmonically convex functions. J. Comput. Anal. Appl. 22(5), 907–922 (2017) MathSciNet
11.
go back to reference Shi, H.N., Zhang, J.: Schur-convexity, Schur-geometric and harmonic convexities of dual form of a class symmetric functions. J. Math. Inequal. 8(2), 349–358 (2014) MathSciNetMATH Shi, H.N., Zhang, J.: Schur-convexity, Schur-geometric and harmonic convexities of dual form of a class symmetric functions. J. Math. Inequal. 8(2), 349–358 (2014) MathSciNetMATH
Metadata
Title
Majorization involving the cyclic moving average
Authors
Tao Zhang
Huan-Nan Shi
Bo-Yan Xi
Alatancang Chen
Publication date
01-12-2018
Publisher
Springer International Publishing
Published in
Journal of Inequalities and Applications / Issue 1/2018
Electronic ISSN: 1029-242X
DOI
https://doi.org/10.1186/s13660-018-1737-4

Other articles of this Issue 1/2018

Journal of Inequalities and Applications 1/2018 Go to the issue

Premium Partner